Anda di halaman 1dari 3

R # Source Ty Question Comments OA Take Aways

* Percent change = Difference / Original * %100


* Check percentages, the sum of bars may not be 100. Then calculate as 25 of 75
; then what of 100 for the percentage questions.
2 Econ-Test IR The number of participants who reported irritability was about 32% greater in ????? than in Group D. * READ
Marine biologist: While we have known that larger marine animals such as whales and dolphins communicate using sound, the fact that smaller marine animals also use sound is a relatively new discovery. These low-frequency sounds produced by animals in the mesopelagic zone--that area between 200 and 1,000 meters below the ocean surface--may be evidence that these smaller animals can commu
You know that it is necessary to find the assumption first, as the fact is
7 Econ-Test IR Indicate two different statements as follows: one statement identifies an assumption required by the marine biologist's argument and the other identifies a possible fact that, if true, would provide significant logical support for the required assumption. Make only two selections, one in each column. dependent on it.
Preconceived notions of the world, especially that of the so-called Third World, have colored our perceptions enough to blind us to its true state, illustrated by decades of reliable statistics. The currently prevalent and popular juxtaposition of the struggling Third World and the flourishing First World is consistent with the state of the world in the mid - twentieth century, not the world in which we curren

With which of the following statements about life expectancy would the author of the passage most likely agree?

A) Life expectancy is inversely proportional to fertility rate.


B) Having a high life expectancy should be a requirement for common consideration as a "First World" country.
C) Life expectancy is an indicator of a country's overall state. C) Very good! In order for the author to use a statistic such as the average life expectancy in a country, that statistic must show something about the country as a whole or else the information would be meaningless.
D) Although life expectancy is currently high in most countries, it will not remain so in those countries commonly referred to as "Third World" countries.
Econ-Test RC E) Low fertility rates cause life expectancy to rise. A) This answer choice incorrectly presumes that there is a relationship between the two factors. However, even in the case the author
C presents, these factors are not necessarily related.
The passage is primarily concerned with
A. presenting various groups views of the motives of those proposing certain legislation
B. contrasting the reasoning of various groups concerning their positions on certain proposed legislation Evaluate : to judge or calculate the quality, importance, amount, or value of
C. tracing the process whereby certain proposed legislation was eventually enacted something
20 D. assessing the success of tactics adopted by various groups with respect to certain proposed legislation
D (OG 89) OG16 verbal1 RC E. evaluating the arguments of various groups concerning certain proposed legislation B Author only states the position of opponenets does not mean evaluation.
16 Diag.Q. PS E If there is a long equation, try to get what is asked exactly.
We see that we can select a boy in 3 ways (since there are 3 boys) and we can select a girl in 2 ways (since there are 2 girls). So we can make a pair in 3*2 = 6 ways.
The basic counting principle deals with problems having distinct spots and available contenders. Here we have 1 spot for a boy and 1 spot for a girl i.e. 2 distinct spots. There are 3 contenders for the empty boy spot and 2 contenders
.com/blog/2011/10/quarter-wit-quarter-wisdom-the-dreaded-combinatorics/
Veritas Blog PS There are 3 boys and 2 girls. We want to select a pair of one boy and one girl for a dance. In how many ways can we do it? Also notice here that it is not 3+2 = 5 ways. This is so because we have to choose a boy AND a girl simultaneously. For every boy, we6 could choose a girl in 2 ways and there are 3 boys so we can choose a pair in 3*2 ways. If we had to cho
2 no, its not even | 3 no, its digits sum to 4 | 5 no, it doesnt end in 0 or 5
7 heres where we may need to get creative. We could perform the long division, but we could also do as we did above with 7 and 11 and find easy to calculate multiples of 7 and work from there. We should know that 2100 is a mult
17 1700 is a multiple of 17; add 340, another multiple, to get closer to 2011 and you have 2040. Subtract 17 and thats 2023, and subtractIf17 youagain
keep toinget
mind
thatthat
range
you(2023
only need
17 =to2006)
test primes;
and recognize
that you
that
only
17have
is nottoa test
factor of 2011. | 19
23 2300 is a multiple of 23. Subtract 230 to get 2070. Subtract 69 to get 2001, add 23 to bracket the range at 2024 and you can tell that 23up is to
notthe
a factor
squareofroot
2011.of |the
29next
2900
highest
is a multiple
square, and
of 29.
that
Subtract
you can290
playtothe
get 2610; do it again to g
37 3700 is a multiple of 37, but far too large, so divide by 2 to get closer to the intended range: 1850. Add 185 to get 2035. Subtract 37 to multiples
establish the
game range
instead
around
of performing
2011 and you longhave
division,
1998,prime
so 37number
is not a testing
factor ofcan2011.
be Lets top and
Man-CAT1 PS Is 2011 prime? 41: 4100 is a multiple of 41, and if we cut that in half were in the range at 2050. Subtract 41 to get 2009 and you can realize that 41 wont be
done
a factor
muchofmore2011.efficiently
| 43:than4300youis amight
multiple
think!
of 43, and if we cut that in half were close at 215
Right triangle PQR is to be constructed in the xy-plane so that the right angle is at P and PR is parallel to the x-axis. The x and y coordinates of P, Q and R are to be integers
Consider
that satisfy
the green
the inequalities
dotted rectangle.
-4<x<5 and
The6<y<16.
x coordinate
How many
varies different
from -4 totriangles
5 and the
with
y coordinate
these properties
varies can
frombe6 constructed?
to 16.
PR has to be parallel to x axis. Since it is a right angled triangle, PQ will be parallel to y axis.
(A) 110 Now lets first fix the vertex P. In how many ways can you choose the coordinates of P? The x coordinate of P can vary from -4 to 5 i.e. it can take 10 values. The y coordinate of P can vary from 6 to 16 i.e. it can take 11 values. In all, the c
(B) 1100 Once vertex P is fixed, the x coordinate of Q will be the same as the x coordinate of P (since PQ is parallel to y axis) and the y coordinate of R will be the same as the y coordinate of P (since PR is parallel to x axis).
(C) 9900 The y coordinate of Q can be chosen in 10 ways. (Out of the 11 values of y coordinate, one is occupied by P so 10 are leftover.)
(D) 10000 The x coordinate of R can be chosen in 9 ways. (Out of 10 values of x coordinate, one is occupied by P so 9 are leftover.)
p.com/blog/2011/09/quarter-wit-quarter-wisdom-use-variations-to-succeed/
Veritas Blog PS (E) 12100 Total number of ways of making the triangle = 110 * 10 * 9 = 9900 C
How many triangles with positive area can be drawn on the coordinate plane such that the vertices have integer coordinates (x,y) satisfying 2 < x < 4 and 5 < y < 7?

(A) 72
(B) 76
(C) 78
(D) 80
p.com/blog/2011/09/quarter-wit-quarter-wisdom-use-variations-to-succeed/
Man-CAT1 PS (E) 84 https://www.veritasprep.com/blog/2011/09/quarter-wit-quarter-wisdom-use-variations-to-succeed/ B
Now that we understand this process, lets examine the symmetry in this. Say we flip the image along the vertical axis what do we get? The figure is still exactly the same, but now the order of cells is reversed to be 4, 3, 2, 1. The pathw
To reach Cell 1, the ball needs to turn left-left-left.
To reach Cell 4, the ball needs to turn right-right-right.
Since the probability of turning left or right is the same, the situations are symmetrical. This will be the same case for Cells 2 and 3. Therefore, Approach:
by symmetry,
Using symetry
we see that:
in symetrical
The probability
possibility
of reaching
questions
Cell 1 = the probability of reaching Cell
Similarly: The probability of reaching Cell 2 = the probability of reaching Cell 3. (There will be multiple ways to reach Cell 2, but the ways of reaching Cell 3 will be similar, too.)
The total probability = the probability of reaching Cell 1 + the probability of reaching Cell 2 + the probability of reaching Cell 3 + the probability
Wayoftoreaching
calculateCell
all4possible
=1 outcomes is :
Because we know the probability of reaching Cells 1 and 4 are the same, and the probabilities of reaching Cells 2 and 3 are the same, this equation (possiblecan
outcome
be written
of anas:event)"
2*(the(number
probability
of events)
of reaching Cell 1) + 2*(the probability of reaching C
Lets find the probability of reaching Cell 1: After the first opening (not the peg, but the opening between pegs 1 and 2 in the first row), the there
ball moves
are 3 left
rows(between
of pegs to pegs
pass,
1 and
each2 row
in second
gives 2row)
ways.
or Thus,
right (between
all possible
pegs 2 and 3 in second r
um/the-figure-shown-represents-a-board-with-4-rows-of-pegs-and-36668.html
PS After that, the ball must move left again the probability of this occurring is also 1/2, since probability of moving left or right is equal. Finally,
outcomes
the ball =must
2*2*2
turn= 2^3=8
left again to reach Cell 1 the probability of this occurring is, again, 1/2. Th
If a rectangle with length units and width units is inscribed in a circle of radius 5 units, what is the value of a + b? 100 if it is too easy to be true, check again
the Mexican Senate recently voted to expand public school sex education programs to kindergarten. This statement indicates that public school sex education programs already exist; the programs cannot be expanded to include a spec
14 Manhattan CAT RC D Expand !!
Premise: There is overcrowding in the schools (not just high schools).
Conclusion: There has been a decline in the reading skills among high school
students.
Assumption: The overcrowding in the schools (in general) has caused the
declining reading skills among high school students.

The correct answer will break the connection between overcrowding in the
schools and declining reading skills among high school students. Eliminate
answers that alter the scope.
B is too broad. The argument is only about high school reading skills. In B, we
Reading skills among high school students in Gotham have been steadily declining, which can only be the result of overcrowding in the schools. don't know which schools have the lower reading scores. The scores could be
Which of the following, if true, most seriously weakens the argument expressed above? lower because of the students in schools other than high school. Eliminate B.
D reverses the scope. Whereas the argument refers to reading skills among high
a) The high school system in Gotham succeeds in giving students a good education at considerably less cost than do most systems. school students, D refers to reading skills in general; whereas the argument
b) Several cities have found that overcrowding in the schools is not always associated with lower reading scores. refers to overcrowding in general -- not just in high school -- D refers to
c) Gotham schools have a greater teacher-to-students ratio than most other school systems. overcrowding only in high schools. Eliminate D.
d) Students' reading skills have not declined in other cities where the high schools are just as crowded as those of Gotham. Only answer choice E breaks the connection between overcrowding in general
http://www.beatthegmat.com/declining-reading-score-t81954.html
CR e) Schools are not overcrowded in many cities where high school reading scores have declined more than they have in Gotham. E and poor reading skills in high school.
Recent advances in surgical procedures have not decreased the average time for recovery from medical operations. Compared to ten years ago, the average post-operative hospital stay for patients undergoing surgery at the ten largest hospitals has actually increased by four days. Although mortality rates for operative procedures are much lower, patients are requiring more time to recover from these p
All of the following, if true, weaken the argument above EXCEPT:
B)Fewer than thirty percent of beds in the largest hospitals are occupied by patients recovering from surgery.
A)Operative and post-operative mortality rates are the most accurate indicators of quality of medical care. -- This choice doesn't weaken or strengthen the conclusion, as the percentage of occupancy does not tell us anything about avg time of post surgical recovery. Author's reasoning is focused around decline in average time of recovery. Jus
B)Fewer than thirty percent of beds in the largest hospitals are occupied by patients recovering from surgery. C)In the past ten years, innovations have allowed previously inoperable conditions to be treated successfully by major surgery.-- This weakens author's conclusion.
C)In the past ten years, innovations have allowed previously inoperable conditions to be treated successfully by major surgery. D)Every year, many surgical procedures that previously required hospital stays are simplified enough that they can safely be performed in outpatient clinics.
D)Every year, many surgical procedures that previously required hospital stays are simplified enough that they can safely be performed in outpatient clinics. -- This weakens author's conclusion and implies that hospital stay would reduce as the surgeries can be performed in clinics.
ub.com/forum/recent-advances-in-surgical-procedures-have-not-decreased-145430.html
CR E)Average surgical recovery time, measured by hospital stay plus time spent disabled from normal activities at home, has decreased by twelve percent in the last ten years.
E)Average surgical recovery time, measured by hospital stay plus time spent disabled from normal activities at home, has decreased B by twelve percent in the last ten years.-- This also clearly weakens author's conclusion

Meteor showers or storms left in the wake of disintegrating comets fall towards the earth and have caused several accidents when they collide with unmanned communication satellites. Irish astronomer James Lawrence postulated a method for calculating the position of the meteor dust at Earth's orbit by studying the dust ejected in 1866 by comet 55P/ Tempel-Tuttle and the Leonid shower return of 1

Which of the following, if true, most seriously weakens the argument above?

1)Predictions about the position of approaching meteor showers are much more accurate today than they were in the last decade of the nineteenth century. Choice C. Evidence other than that used by Lawrence has previously enabled scientists to predict the position of the meteor dust as it approaches the earth.
2)Scientists have corroborated Lawrences work and established the significance of the Leonid shower to calculations of the position of meteor dust. Premise: This method allowed him to predict with reasonable accuracy the position of the meteor dust as it approaches the earth.
3)Evidence other than that used by Lawrence has previously enabled scientists to predict the position of the meteor dust as it approaches the earth. Conclusion: accidents caused by the collision of meteor showers with unmanned communication satellites will be prevented
4)Scientists have not been able to understand exactly how the meteor showers disintegrate before they reach the earth, yet manage to have a significant impact on the unmanned satellites.
CR 5)Some scientists have found several discrepancies between Lawrences work with meteors and his earlier work. If accidents caused by the collision of meteor showers with unmanned communication satellites will be prevented because this method
c allowed him to predict with reasonable accuracy the position of the meteor dust as it approaches th
In Colorado subalpine meadows, nonnative dandelions co-occur with a native flower, the larkspur. Bumblebees visit both species, creating the potential for interactions between the two species with respect to pollination. In a recent study, researchers selected 16 plots containing both species; all dandelions were removed from eight plots; the remaining eight control plots were left undisturbed. The con
The scientists concluded that "the presence of dandelions facilitates pollination (and hence seed production) in the native species by attracting more pollinators to the mixed plots." We want to weaken this conclusion.
Which of the following, if true, most seriously undermines the researchers reasoning? (A) If all the bees were visiting dandelions, then this would not explain why the larkspurs got so pollinated, producing a great quantity of seeds. This is simply inconsistent with the evidence. This is not correct.
(B)This is a strengthener. If this is true, then it would explain why having dandelions in the plot would result in more pollination for the larkspurs. This is a typical GMAT CR trap, having a strengthener for a weakener, or vice versa. This is
(A) Bumblebees preferentially visit dandelions over larkspurs in mixed plots. (C) This is a problems down the road, a reason why in the big picture dandelions might be a problem for larkspars, but it doesn't do anything to address the issue of which plants get pollinated and how many seeds are produced. This is n
(B) In mixed plots, pollinators can transfer pollen from one species to another to augment seed production. (D) Even if this is true, it is not relevant, because in terms of the experiment, only seed production was measured. There was no measurement of seed germination: at most, it was merely inferred from seed production. The experiment l
(C) If left unchecked, nonnative species like dandelions quickly crowd out native species. (E) This may appear irrelevant at first glance, but think about it. The control plot (with dandelions) produced more seeds than the experimental, dandelion-free plot. How was that latter plot prepared? The prompt says: "all dandelions w
(D) Seed germination is a more reliable measure of a species fitness than seed production.
OG CR (E) Soil disturbances can result in fewer blooms, and hence lower seed production. (E) has to be the OA. This is a brilliant question, because at first glance, it may appear that (E) is entirely out-of-scope and irrelevant.e You have to think about the details of the prompt to recognize why it is so relevant.
Due to an easing of state sanctions against hunting, Deersdale Preserve has experienced an influx of hunters in the last several months. During this time, the silvertail fox, a popular target for hunters, has seen a marked decrease in population. Therefore, to prevent the population of silvertail foxan important apex predator responsible for keeping the Deersdale Countys rabbit population in checkfro

Which of the following, if true, may suggest that stricter sanctions against hunting would not have the desired effect?

A.The population of rabbits has surged ever since the hunting sanctions in Deersdale County were lifted.
B.The silvertail fox population varies greatly throughout the year, especially during winter when prey becomes scarce.
C.The local authorities are expecting even more hunters in the coming year to arrive to the park. (B) would be true in normal circumstances. That's part of the point. It's nothing out of the ordinary cycle of seasons. Lele wrote this question so that most users would pass over all five answers on the first go, and then have to come back
D.The silvertail fox had been experiencing a population surge shortly before the state sanctions against hunting were eased.
Magosh CR E.The grey wolf, a large predator that competes with the silvertail fox over Deersdale Preserves rabbit population, has seen its numbers decrease since the arrival of the We
hunters.
need something, anything, that would suggest that a decline in the silvertail fox is not due solely to the increased hunting. Well,b winter also reduces the fox population. Is the question happening in winter, or is winter imminent? Har
An independent analyst asserts that the new Pokia digital camera for mobile phones is more precise, of higher quality and costs less than any of the popular competing models. As a result, Pokia will become the more desirable, low-priced alternative to currently existing models of digital cameras.

Which of the following, if true, would most challenge the argument above?

(A) Many retailers already carry one or more low-priced digital cameras and are disinclined to carry another.
(B) Several lower-priced models of digital cameras will soon be introduced by other mobile phone manufacturers.
(C) The Pokia Corporations digital camera can be used in conjunction with higher-priced mobile phones manufactured by other companies. This is a weaken question. The analyst asserts that it is more precise, of higher quality, and less expensive than other options. Well cost and precision are not very subjective, but quality certainly is. If, as in answer choice E, the analysts' o
(D) Most of the individuals and companies that would be expected to make up the potential market for the Pokia digital camera have already fulfilled their digital camera needs.
http://gmatclub.com/forum/pokia-digital-camera-163310.html
CR (E) The independent analyst whose views were incorporated in the statement above has used measures of quality that are not universally accepted by the consumer public. Answer choices A and D are both incorrect because they each focus only on the practical success of the camera, and not on whether e or not it is a preferred alternative. If many retailers refuse to carry the camera, as in A, then sure that m
The recent decline in the value of the dollar was triggered by a prediction of slower economic growth in the coming year. But that prediction would not have adversely affected the dollar had it not been for the governments huge budge
Which of the following, if true, would most seriously weaken the conclusion about how to prevent future currency declines?
Which argument we are trying to weaken here :
governments huge budget deficit, which must therefore be decreased to prevent future currency declines.

The recent decline in the value of the dollar was triggered by a prediction of slower economic growth in the coming year. But that prediction would not have adversely affected
(A) Thethe
government
dollar hadhas
it not
made
been
little
forattempt
the governments
to reduce the
hugebudget
budgetdeficit.
deficit,
(This
which
does
must
nottherefore
provide evidence
be decreased
that budget
to prevent
deficit
future
was currency
the only reason)
declines.

Which of the following, if true, would most seriously weaken the conclusion about how to prevent future currency declines? (B) The budget deficit has not caused a slowdown in economic growth.(Not relevant in the argument here)

(A) The government has made little attempt to reduce the budget deficit. (C) The value of the dollar declined several times in the year prior to the recent prediction of slower economic growth. (This does not talk about the budget deficit was present at these times of decline or not)
(B) The budget deficit has not caused a slowdown in economic growth.
(C) The value of the dollar declined several times in the year prior to the recent prediction of slower economic growth. (D) Before there was a large budget deficit, predictions of slower economic growth frequently caused declines in the dollars value. (This directly concludes that large budget deficit is not responsible for the decline in the dollar's value w
(D) Before there was a large budget deficit, predictions of slower economic growth frequently caused declines in the dollars value.
lub.com/forum/the-recent-decline-in-the-value-of-the-dollar-was-triggered-130234.html
CR (E) When there is a large budget deficit, other events in addition to predictions of slower economic growth sometimes trigger declines in currency value. (E) When there is a large budget deficit, other events in addition to predictions of slower economic growth sometimes trigger declines
d in currency value. (This infers that " governments huge budget deficit, which must therefore be decr
The price the government pays for standard weapons purchased from military contractors is determined by a pricing method called historical costing. Historical costing allows contractors to protect their profits by adding a percentage increase, based on the current rate of inflation, to the previous years contractual price.

Which of the following statements, if true, is the best basis for a criticism of historical costing as an economically sound pricing method for military contracts?

(A) The government might continue to pay for past inefficient use of funds. (A) The government might continue to pay for past inefficient use of funds.
(B) The rate of inflation has varied considerably over the past twenty years. This correctly shows that bad deals in the past will continue to affect contracts in the future. Pick A.
(C) The contractual price will be greatly affected by the cost of materials used for the products.
(D) Many taxpayers question the amount of money the government spends on military contracts. (B) The rate of inflation has varied considerably over the past twenty years.
CR GMAT
Official Guide for (E) The pricing
Verbal method2nd
Review, based on historical costing might not encourage the development of innovative weapons.
Edition if the rate of inflation varies, it will still increase the cost either by a little or alot. this doesnt say anything about historical prices. a
A system-wide county school anti-smoking education program was instituted last year. The program was clearly a success. Last year, the incidence of students smoking on school premises decreased by over 70 percent.

Which of the following, if true, would most seriously weaken the argument in the passage?
(A) The author of this statement is a school system official hoping to generate good publicity for the anti-smoking program. (B) Most students who smoke stopped smoking on school premises last year continued to smoke when away from school.
(B) Most students who smoke stopped smoking on school premises last year continued to smoke when away from school. Incorrect - this statement is a bit neutral. It neither strengthens the argument nor necessarily weakens it. Furthermore, if the ultimate goal of the program was to decrease on school incidences, then it was still a success.
(C) Last year, another policy change made it much easier for students to leave and return to school grounds during the school day.
(D) The school system spent more on anti-smoking education programs last year than it did in all previous years. (C) Last year, another policy change made it much easier for students to leave and return to school grounds during the school day.
CR (E) The amount of time students spent in anti-smoking education programs last year resulted in a reduction of in-class hours devoted to academic subjects. Correct - We needed something that indicated another motivation outside of the program. This fulfills that necessity and weakens the
c argument. If we can positively associate the decrease in on-school incidences to another policy, then
Degler does more than merely summarizing existing research; he constructs a coherent picture of two centuries of studies dealing with the changing roles of women.
(A) Degler does more than merely summarizing (A) Degler does more than merely summarizing.summarizing and constructs do not go //
(B) Deglers study is more than a mere summarizing of (B) Deglers study is more than a mere summarizing of ---- summarizing of and constructs do not go well; one is a gerund and the other is a verb
(C) Degler has done more than a mere summarizing of (C) Degler has done more than a mere summarizing of ----- same as B he does summirize <> he constructs
(D) Deglers study has done more than summarize merely (D) Deglers study has done more than summarize merely- comparing a study with he a person; unparallel ile ayn deil
SC (E) Degler does more than merely summarize (E) Degler does more than merely summarize ------- Degler tallies with He, does summarize tallies with constructs --- correct e he does summarizing <> he constructs
As accelerating climate change and other man-made environmental degradations create growing alarm across the planet, the Sami people have much to teach the world about how to adapt, survive, and thrive.
First, we split the sentence in terms of meaning. What creates growing alarm across the planet? Only the man-made environmental degradations or both climate change and them? The intended meaning should be "the climate change a
A. As accelerating climate change
B. As accelerated climate change Between A and B, should we use V+ing or V+ed form. To come up with an answer, let think: climate change accelerates by itself or It is accelerated? Clearly, it accelerates itself ==> V+ing form is correct.
C. As climate change accelerates
D. As climate change is accelerated a) The climates changes accelerated by the soil erosion has become a havoc for the farmers of the Indus valley. -> What caused the acceleration in climate changes = soil erosion
SC E. As climate changes are accelerated b) The accelerating car hit the truck before plunging into the valley. -> There is no mention in the sentence who caused the acceleration,
e since the car was accelerating by itself and the word accelerating just adds additional information.
A.hardy, disease-resistant and yields
Wrong. Because "hardy" || disease-resistant, so there should be "and" between "hardy" and "disease-resistant" to make the parallel structure clear.
Agricultural scientists continue to search for a strain of wheat that is hardy, disease-resistant and yields more grain per acre. B.hardy, resists disease, and that yields
Wrong. If "that is"|| resists || yields ==> the second "that" is wrong. The correct structure should be: a strain of wheat that is hardy, resists disease and yields.....
A.hardy, disease-resistant and yields C.hardy and disease-resistant and that yields Correct.
"hardy" || "disease-resistant;
B.hardy, resists disease, and that yields "that is ...." || "that yields
D.hardy and resist disease, yielding
C.hardy and disease-resistant and that yields Wrong. "a strain" is singular ==> "resist" is wrong. Should be "resists".
E.hardy, disease-resistant, and yielding
D.hardy and resist disease, yielding Wrong. Although the structure is parallel, the meaning is wrong. "Hardy" and "disease-resistant" are characteristics of the strain of wheat, but "yielding" is the advantage of the grains, so we cannot put them together in a parallel structu
- My professor who is tall, black-hair and who teaches math is very smart. Agricultural scientists continue to search for a strain of wheat that is hardy and
SC E.hardy, disease-resistant, and yielding - We can put "tall" and "black-hair" in one parallel structure, but cannot put "tall" + "black-hair" + "teach math". They are in different
c categories.
resists disease, yielding more grain per acre. >>>> RIGHT
R # Source Ty Question Comments OA Take Aways
The core sentence is :
The Federal investigators at Stapleton Industries have failed to find any evidence for 2 things :
a)the unusually large contributions to its accounts are derived from government kickbacks
b)its officers are guilty

A has parallelism error and verb tense error: has suggested is incorrect. The prosecutors have failed , it's results are out. So don't need perfect tense here. The parallelism error :
that has suggested blah blah....nor its officers guilty [not parallel]

B) parallelism error:
1 ) that suggests that the unusually large contributions to its accounts are derived from government kickbacks" is a that clause. Two uses of that are probably not correct.
2)its officers guilty - this is NOT.

The Federal investigators at Stapleton Industries have failed to find any evidence that has suggested that the unusually large contributions to its accounts are derived from
C)suggesting
government that
kickbacks,
the unusually
nor itslarge
officers
contributions
guilty of improper
to its accounts
relations
hadwith
beenindustry
derivedregulators.
from government kickbacks, with its officers guilty
(A) that has suggested that the unusually large contributions to its accounts are derived from government kickbacks, nor its officers guilty Screws up the original meaning by using the -Ing modifier.
(B) that suggests that the unusually large contributions to its accounts are derived from government kickbacks, or its officers guilty
(C) suggesting that the unusually large contributions to its accounts had been derived from government kickbacks, with its officers guilty E)Parallelism Error:
(D) to suggest that the unusually large contributions to its accounts are derived from government kickbacks or that its officers are guilty 1)to suggest that the unusually large contributions to its accounts had been derived from government kickbacks,
orum/2-the-federal-investigators-at-stapleton-industries-have-154221.html
Magoosh SC (E) to suggest that the unusually large contributions to its accounts had been derived from government kickbacks, nor were its officers guilty 2)nor were its officers guilty d
The General Agreement on Trade & Tariffs was a landmark deal in committing at its signing in 1973 to reduce trade restrictions around the world.

in committing at its signing in 1973 to reduce


by committing at its signing in 1973 on the reduction of
at its 1973 signing for committing to reduce
for the commitment at its 1973 signing to reduce
SC when it was signed in 1973 for its commitment to reduce d
The push by rich countries to cut emissions have raised fears that it is an attempt to secure new markets for environmental technologies.

have raised fears that it is an attempt have raised fears that it is an attempt
have raised fears that they are trying have raised fears that they are trying
has raised fears that they are trying has raised fears that they are trying --> in ithe above sentence "Push" is the subject ,thus they cannot refer to it and the remaining plural nouns are countries and emmisions thus there is an ambiguity .
has raised fears that it is their attempt has raised fears that it is their attempt It / they must refer ONE thing in the sentence. If there are several possibilities,
SC has raised fears that it is an attempt has raised fears that it is an attempt ---> it correctly refers to "Push" d then the sentence is wrong
Some people believe that gaining wealth is due to luck. However, research from many institutions worldwide indicates a strong correlation between gaining wealth and high educational levels. Thus research supports the view that gaining wealth is largely the result of making informed career and business choices.

The reasoning in the argument is most vulnerable to criticism on the grounds that the argument ______________
The argument contains a causal conclusion that asserts that good wealth is primarily caused by informed career and business choices (education) the conclusion is that research supports the view that gaining wealth is largely the result o
A. presumes, without providing justification that only highly educated people make informed career and business choices. -Does the argument presume that to make an informed lifestyle choice a person must be highly educated? The author certainly believes that high educational levels lead to informed career and business choices, but the answer suggests
B. overlooks the possibility that people who make informed career and business choices may nonetheless belong to a poor family. -The author specifically notes that gaining wealth is largely the result of making informed career and business choices. There is no mention of poor family background, nor is needed because the argument focuses on a correlation betwee
C. presumes, without providing justification, that informed career and business choices are available to everyone. -The author does not make the presumption that informed career and business choices are available to everyone, just that making good choices generally results in gaining wealth.
D. overlooks the possibility that the same thing may causally contribute both to education and to gaining wealth. -Correct. Note that one of the errors is assuming that no third element caused both the stated cause and the stated effect. This answer choice indicates that a third element could cause both the conditions described in the argument. No
CR E. does not acknowledge that some people who fail to make informed career and business choices have gained wealth. -The author specifically says that the effect is largely the result of the cause, and that statement indirectly allows other causes to lead
D to the effect, even if one does not make an informed career and business choices.
A bus leaves city M and travels to city N at a constant speed, at the same time another bus leaves city N and travels to city M at the same constant speed. After driving
for 2 hours they meet at point P. The following day the buses do the return trip at the same constant speed. One bus is delayed 24 minutes and the other leaves 36
minutes earlier. If they meet 24 miles from point P, what is the distance between the two cities?

A. 48
B. 72
C. 96
D. 120
GMAT Club Tests PS E. 192 e
If N=1234@ and @ represents the units digit of N, is N a multiple of 5? 1234@1234@ to be divisible by 5, symbol "@@" should represent either 0 or 5. So the question asks whether @@ equals to 0 or 5.
(1) @!@! is not divisible by 5. @ can be 0, 1, 2, 3, or 4 (note that 0!=1). Not sufficient. 0 is divisible by every integer except 0 (0/5 OK - 0/0 NO - 7/0 UNDEFINED)
(1) @! is not divisible by 5 (2) @@ is divisible by 9. @ can be 0 or 9 (note that zero is divisible by every integer except zero itself). Not sufficient.
GMAT Club Tests DS (2) @ is divisible by 9 (1)+(2) Intersection of the values for @@ from (1) and (2) is @=0@=0. Sufficient. C 0!=1
Notice that since events AA and BB are independent, then the probability that both occur, equals to the product of their individual probability, so P(A and B)=P(A)P(B)P(A and B)=P(A)P(B). Also notice that 0P(A)1 and 0P(B)1
If Event AA and Event BB are independent, is the probability that both Event AA and Event BB will happen greater than 0.3? how can the probability that both Event A and Event B will happen be more than
(1) Probability that AA will happen is 0.25 (1) Probability that AA will happen is 0.25. Now, since P(A)=0.25, then P(A and B)=P(A)P(B)0.25<0.3. Sufficient. Or consider the following: individual
how can the probability
probability that both
of each Event AA
happening? So,and
theEvent BB willthat
probability happen
bothbe more than individual p
happen
GMAT Club Tests DS (2) Probability that BB will NOT happen is 0.71 (2) Probability that B will NOT happen is 0.71. The same here: since P(B)=10.71=0.29, then P(A and B)=P(A)P(B)0.29<0.3. Sufficient.
d cannot be more than Event A (or B) happen individually
The numbers {1, 3, 6, 7, 7, 7} are used to form three 2-digit numbers. If the sum of these three numbers is a prime number pp, what is the largest possible value of pp?
A. 97
B. 151 The factorial of a negative number is undefined.
C. 209 0!=1.
D. 211 Only odd factorials: 0!=1 and 1!=1.
GMAT Club Tests PS E. 219 What is the largest possible sum of these three numbers that we can form? Maximize the first digit: 76+73+71=220=even76+73+71=220=even,
d Onlysoprime
not afactorial
prime. Let's
: 2!=2.
try next largest sum, switch digits in 76 and we'll get: 67+73+71=211=pr
Is x the square of an integer? The question basically asks whether xx is a perfect square (a perfect square, is an integer that is the square of an integer. For example 16=4216=42, is a perfect square).

Perfect square always has even powers of its prime factors. The reverse is also true: if a number has even powers of its prime factors then it's a perfect square. For example: 36=223236=2232, powers of prime factors 2 and 3 are even
(1) When x is divided by 12 the remainder is 6.
(1) When xx is divided by 12 the remainder is 6. Given that x=12q+6=6(2q+1)=23(2q+1)x=12q+6=6(2q+1)=23(2q+1). Now, since 2q+12q+1 is an odd number then the power of 2 in xx will be odd (1), thus xx cannot be a perfect squar
GMAT Club Tests DS (2) When x is divided by 14 the remainder is 2. (2) When xx is divided by 14 the remainder is 2. Given that x=14p+2x=14p+2. So, xx could be 2, 16, 30, ... Thus, xx may or may not be
A a perfect square. Not sufficient.
If a circus were to sell all of its 220 tickets for this month's performance at its usual price, the revenue from sales would be 10% greater than that collected last month.
If the circus were to raise the ticket price by 5% and sell 200 tickets, what percent less would last month's revenue be compared to this month's revenue?

A. 2
B. 5
C. 100/21
D. 110/20
E. 9/4
GMAT Club Tests PS http://gmatclub.com/forum/m25-184435.html C Percent = ( change / Original ) * 100
Is abc divisible by 32?
0 is divisible by any number
(1) a, b, and c are consecutive even integers
GMAT Club Tests DS (2) ac<0 http://gmatclub.com/forum/m15-184062.html C 0/ 32 >>> divisible by 32
If a and b are two-digit positive integers greater than 10, is the remainder when a is divided by 11 less than the remainder when b is divided by 11?
(1) The remainder when aa is divided by 69 is the fifth power of a prime number.
GMAT Club Tests DS (2) The remainder when bb is divided by 12 is b http://gmatclub.com/forum/m30-184576.html D READ CAREFULLY
If for any positive integer x, d[x] denotes its smallest positive odd divisor and D[x] denotes its largest odd divisor, is x even?
(1) D[x]d[x]=0
GMAT Club Tests DS (2) D[3x]=3 http://gmatclub.com/forum/m21-184291.html E READ CAREFULLY - Tells odd
Is the product of two positive numbers x and y larger than their sum?
(1) x and y are integers
GMAT Club Tests DS (2) y>x>1 C Think of integers and non-integers always
If nn is a positive integer is n"2n divisible by 12 ?

(1) n is divisible by 11.


GMAT Club Tests DS (2) n is divisible by 19. http://gmatclub.com/forum/m23-184340.html E Think of multiplies of 11 including 11*12
The product of three distinct positive integers is equal to the square of the largest of the three numbers, what is the product of the two smaller numbers?

(1) The average (arithmetic mean) of the three numbers is 34/3.


GMAT Club Tests DS (2) The largest number of the three distinct numbers is 24. http://gmatclub.com/forum/m27-184484.html D
(1) |x2|<2y|x2|<2y. The left hand side of the inequality is an absolute value, so the least value of LHS is zero, thus 0<2y0<2y, or y<2y<2 (if yy is more than or equal to 2, then y20y20 and it cannot be greater than |x2||x2|)

If x and y are positive integers, is x a prime number? So, we have that: |x2|<1|x2|<1, which implies that 1<x2<11<x2<1, or 1<x<31<x<3, thus x=2=primex=2=prime. Sufficient.
(1) |x2|<2y|
GMAT Club Tests DS (2) x+y3=|1y| (2) x+y3=|1y|x+y3=|1y|. Since yy is a positive integer, then 1y01y0, thus |1y|=(1y)|1y|=(1y). So, we have that x+y3=(1y)x+y3=(1y),
D which gives x=2=primex=2=prime. Sufficient.
A jar contains 8 red marbles and y white marbles. If Joan takes 2 random marbles from the jar, is it more likely that she will have 2 red marbles than that she will have
one marble of each color? While calculating probability, don't forget the multiply with number of ways.
i.e: there are 2 ways he can have red and white marble
(1) y 8 RED and WHITE
Q-12Manhattan CAT2 DS (2) y 4 http://gmatclub.com/forum/a-jar-contains-8-red-marbles-and-y-white-marbles-if-joan-101748.html B WHITE and RED
(1) INSUFFICIENT: Since this equation contains two variables, we cannot determine the value of y. We can, however, note that the absolute value expression | x 2 4| must be greater than or equal to 0. Therefore, 3| x 2 4| must b

(2) INSUFFICIENT: To solve this equation for y, we must consider both the positive and negative values of the absolute value expression: When there is aboslute value on left side, thinking of positive can be enough for
If 3 y > 0, then 3 y = 11 y = -8 right side
If 3 y < 0, then 3 y = -11 y = 14
What is the value of y? Since there are two possible values for y, this statement is insufficient. the absolute value expression | x 2 4| must be greater than or equal to 0.
Therefore, 3| x 2 4| must be greater than or equal to 0, which in turn means
(1) 3| x 2 4| = y 2 (1) AND (2) SUFFICIENT: Statement (1) tells us that y is greater than or equal to 2, and statement (2) tells us that y = -8 or 14. Of the two possible
that y values,
2 mustonly
be greater
14 is greater
than or
than
equal
or equal
to 0. to
If y2.Therefore,
2 > 0, thenthe
y >two
2. statements together tell u
Q-25Manhattan CAT2 DS (2) |3 y| = 11 C
A certain square is to be drawn on a coordinate plane. One of the vertices must be on the origin, and the square is to have an area of 100. If all coordinates of the
vertices must be integers, how many different ways can this square be drawn?
4
6
8
10 http://gmatclub.com/forum/a-certain-square-is-to-be-drawn-on-a-coordinate-plane-127018.html
12
Q-26Manhattan CAT2 PS As for your question I doubt that this is a realistic GMAT question. Though if you find that # of squares should be multiple of 4 you'll be left with A, C and E choices right away. Next, you can also rule out A as at least 2
Owner of JavaJoint: Over the past year, the coffee store has become a daily hang-out for more and more teenagers. Many of our adult customers do not appear comfortable with this kind of crowd and some of them have told me that they will no longer stop here for a coffee drink. Since my goal is to maximize our revenue, I want you to discourage teenagers from coming here and start cultivating a mo
Store manager: Are you sure? On average, each teenager spends just as much as the average adult does, and we have far more new customers than we have lost over the(A)past
Theyear.
store manager makes no attempt to question the veracity of the statements of the owner. When the manager asks Are you sure? he is questioning the owners directive (or its effectiveness), not the veracity of the owners e

The store manager responds to the owner by _____ (B) The implementation of the owners directive is not mentioned by the manager and is not relevant.

(A) questioning the veracity of owners evidence (C) CORRECT. By asserting that the average teenager spends just as much money as the average adult and that there are more new customers than former customers, the manager is presenting new evidence that implies that the curren
(B) arguing that it would be difficult to implement the owners directive
(C) offering new evidence implying that the status quo is not incompatible with the owners goal (D) The manager only states that the average teenager spends as much as the average adult; hence, he makes a claim as to the relative revenue generated by the average member of each group. He does not make any claim as to the r
(D) demonstrating that the average teenage customer is as profitable as the average adult customer
V-22 Manhattan CAT2 CR (E) offering new evidence refuting that presented by the owner (E) The new evidence that the manager presents neither contradicts nor refutes any of the evidence that the owner had previous presented.
C
Club X has more than 10 but fewer than 40 members. Sometimes the members sit at tables with 3 members at one table and 4 members at each of the other tables,
and sometimes they sit at tables with 3 members at one table and 5 members at each of the other tables. If they sit at tables with 6 members at each table except one
and fewer than 6 members at that one table, how many members will be at the table that has fewer than 6 members?

(A) 1
(B) 2
(C) 3
(D) 4
m/club-x-has-more-than-10-but-fewer-than-40-members-sometimes-143987.html
Gmat Club PS (E) 5 E
Focus on any one of the two elements, say Copper (or Nickel if you fancy it more. Either way, you will get the same answer). Forget about the other one. If you notice, in the question above there are two elements in each mixture too (o
There are 2 bars of copper-nickel alloy. One bar has 2 parts of copper to 5 parts of nickel. The other has 3 parts of copper to 5 parts of nickel. If both bars are melted First bar is 2/7th copper (2 parts copper and 5 parts nickel to get a total of 7 parts). Second bar is 3/8th copper and the combined alloy is 5/16th copper.
together to get a 20 kg bar with the final copper to nickel ratio of 5:11. What was the weight of the first bar?
Now, lets calculate the ratio of weights of the two bars.
(A) 1 kg w1/w2 = (A2 Aavg)/(Aavg A1)
(B) 4 kg w1/w2 = (3/8 5/16)/(5/16 2/7) = 7/3
(C) 6 kg
(D) 14 kg So weight of first bar: weight of second bar = 7:3
om/blog/2011/04/quarter-wit-quarter-wisdom-dont-get-mixed-up-in-mixtures/
Veritas PS (E) 16 kg Out of a total of 20 kgs, the weight of the first bar was 14 kg and the weight of the second bar was 6 kg. D
Remaining numbers to fill last two digits (3,4,6,8,9): Total 5
A telephone number contains 10 digit, including a 3-digit area code. Bob remembers the area code and the next 5 digits of the number. He also remembers that the
remaining digits are not 0, 1, 2, 5, or 7. If Bob tries to find the number by guessing the remaining digits at random, the probability that he will be able to find the Probability of choosing right numbers in two places = 1/5 * 1/5 = 1/25
correct number in at most 2 attempts is closest to which of the following ? Probability of not choosing right numbers in two places = 1-1/25 = 24/25
--
A. 1/625 At most two attempts: 1) Wrong-1st Attempt, Right - 2nd, 2) Right - 1st Attempt
B. 2/625 1) = 24/25 * 1/25 = 24/625
C. 4/625 2) = 1/25
D. 25/625
orum/a-telephone-number-contains-10-digit-including-a-3-digit-59697.html
Gmat Club PS E. 50/625 Add 1 and 2, 24/625 + 1/25 = 49/625 ~ 50/625 E
Every day a certain bank calculates its average daily deposit for that calendar month up to and including that day. If on a randomly chosen day in June the sum of all
deposits up to and including that day is a prime integer greater than 100, what is the probability that the average daily deposit up to and including that day contains
fewer than 5 decimal places?

(A) 1/10
(B) 2/15
(C) 4/15
(D) 3/10
m/forum/every-day-a-certain-bank-calculates-its-average-daily-97456.html
Gmat Club PS (E) 11/30 D contains fewer than 5 decimal places = terminating decimal
R # Source Ty Question Comments OA Take Aways
A woman sold 100 oranges at $12.10, some at the rate of 3 for 35 cents and the rest at 7 for 85 cents. How many were sold at the first rate?

A. 45
B. 21
C. 9
D. 15
orum/a-woman-sold-100-oranges-at-12-10-some-at-the-rate-of-98032.html
Gmat Club PS E. 12 C Check answers
Randy is going to flip a coin x times. What is the minimum value of x for which the probability that he flips all heads is less than 0.01%?

A. 10
B. 11
C. 12
D. 13
randy-is-going-to-flip-a-coin-x-times-what-is-the-minimum-value-of-x-194527.html
Gmat Club PS E. 14 E
If r + s > 2 t, is r > t ?

(1) t > s
16 Manhattan CAT3 DS (2) r > s D Think simple, combine in equalities.
We can approach this problem in two ways. The first, and probably easier, method is to work backward from the answer choices and calculate how much Plans A and B pay out in each case. When the two calculated amounts are the sam

Health insurance Plan A requires the insured to pay $1,000 or 50% of total cost, whichever is lower. Plan B requires the insured to pay the initial $300, but then pays An algebraic approach is the other possibility. First we need to determine how much Insurance Plan A pays. The question states that the insured pays either $1,000 or 50% of the total cost, whichever is lower. Let x be the total cost. If th
80% of the cost over $300. Which of the following is a cost level for which both insurance plans pay out the same amount?
$600 For Insurance Plan B, the insured pays the first $300, and the insurance plan pays 80% of everything over $300. So Insurance Plan B pays 0.8(x - 300).
$1,000 Because there are two possible payment structures for Plan A, so we need to set up 2 equations. We are looking for the cost level for which both plans pay out the same amount, so we can set the two plans equal to each other. The two
$3,800 0.8(x 300) = x 1,000 OR 0.8(x 300) = .5x
$5,300 Solving the first equation gives us x = 3,800, and solving the second equation gives us x = 800.
25 Manhattan CAT3 PS $6,200 $800 isn't one of our options, but $3,800 is. C Understand the meaning
If x2 < x and x is written as a terminating decimal, does x have a nonzero hundredths digit?
Think of all variations
(1) 10x is not an integer. 0,a
0,ab
25 Manhattan CAT3 DS (2) 100x is an integer. C 0,abc
12 people will be selected from a pool of 15 people: 10 men (2/3 of 15) and 5 women (1/3 of 15). The question asks for the probability that the jury will comprise at least 2/3 men, or at least 8 men (2/3 of 12 jurors = 8 men).
The easiest way to calculate this probability is to use the 1-x shortcut. The only way the jury will have fewer than 8 men is if a jury of 7 men and 5 women (the maximum number of women available) is selected. There cannot be fewer

The total number of juries that could be randomly selected from this jury pool is: 5 Green 8 white balls
15! / 12!3! = 455
If a jury of 12 people is to be selected randomly from a pool of 15 potential jurors, and the jury pool consists of 2/3 men and 1/3 women, what is the probability that The number of ways we could select 7 men from a pool of 10 men is: we will choose 5 balls. what is the probability that there will be 3 green balls
the jury will comprise at least 2/3 men? 10! / 7!3! = 120
The number of ways we could select 5 women from a pool of 5 women is: - 3 green balls means 2 WB 0= 3 + 2 = 5 balls in total
24/91 5!/5! = 1
45/91 - Calculate choosing 3 out of 5 green balls
2/3 This makes practical sense, in addition to mathematical sense. All of the women would have to be on the jury, and there is only one way that
- Calculate
can happen.
choosing
Putting2 out
these
of selections
8 white balls
together, the number of ways a jury of 7 men and 5 wom
67/91 The probability that the jury will be comprised of fewer than 8 men is thus 120/455 = 24/91. - Multiply them to get the # of ways having 3G + 2 W balls
28 Manhattan CAT3 PS 84/91 Therefore, the probability that the jury will be comprised of at least 8 men is 1 (24/91) = 67/91. D - The rest is classical probability
Jane can paint the wall in J hours, and Bill can paint the same wall in B hours. They begin at noon together. If J and B are both even numbers is J=B?
they painted the wall in 288 minutes or in 288/60 hours.
(1) Jane and Bill finish at 4:48 p.m.
forum/jane-can-paint-the-wall-in-j-hours-and-bill-can-paint-the-87131.html
Gmat Club DS (2) (J+B)^2=400 http://gmatclub.com/forum/jane-can-paint-the-wall-in-j-hours-and-bill-can-paint-the-87131.html A = In one hour, working together, they painted 60/288 of the wall.
My solution: (From the diagram table) we have to know, is a/c > (a+b)/(c+d) ?
or, ac+ad>ac+bc or, ad>bc ?
Inequalities can be multiplied.
st(1), a+b/ c+d >b/d
or, ad+bd>bc+bd a>b
or, ad>bc so st(1) alone is sufficient. d>c
a*d > b*c
Each employee of Company Z is an employee of either division X or division Y, but not both. If each division has some part time employees, is the ratio of the number st(2), we know a>b
of full-time employees to the number of part-time employees greater for division X than for Company Z? and d>c . X=a+b
Multiplying the both equation gives us, ad>bc. so st(2) alone is sufficient too. Y=c+d
(1) the ratio of the number of full time employees to the number of part-time employees is less for division Y than for Company Z In the weighted average if ratio (full time / part time) increases in division X, the ratio automatically decreases in division Y to keep the weighted
Z = (a+c)
average
4 ( b+(ratio)
d) ---->>>
samea+c
for/ company
b+d mustZ.beThis
somewher
reasoning
between
is for Statement
a/b and c/d.
2)

(2) More than half of the full-time employees of Company Z are employees of Division X, and more than half of the part-time employees of Company Z are employees Thus both statements individually are sufficient to solve the problem. %Full-Z will be a weighted average of %Full-X and %Full-Y. So either X>Z>Y or
ach-employee-of-company-z-is-an-employee-of-either-division-54083.html?kudos=1
Gmat Club DS of Division Y Thus Answer is (D) D X<Z<Y, Z will always be in the middle.
Say a meal consists of a sandwich and a milkshake. You eat 1/2 of the sandwich and drink 1/2 of the milkshake. Can I say you have had 1/2 of the meal? Sure, right?
If you eat only 1/4 of the sandwich and drink 1/4 of the milkshake, then you would have had only 1/4 of the meal.
What happens in case you eat 1/2 of the sandwich but drink only 1/4 of the milkshake? In that case, you have had less than 1/2 of the meal but certainly more than 1/4 of the meal, right?

Each piglet in a litter is fed exactly one-half pound of a mixture of oats and barley. The ratio of the amount of barley to that of oats varies from piglet to piglet, but So when piglet A is fed 1/4 of the Oats and 1/6 of the Barley, it is fed less than 1/4 of the total food but more than 1/6 of the total food. number of piglets = n
each piglet is fed some of both grains. how many piglets are there in the litter? Since it is given that each piglet gets the same amount of food and there are n
Another thing to consider here is that number of piglets has to be a positive integer, say 'n'. Now, since it is given that each piglet gets the same
piglets,
amount of food and there are n piglets, each piglet will get 1/n of the total food. So piglet A
(1) Piglet A was fed exactly 1/4 of the oats today each piglet gets 1/n of the total food.
m/each-piglet-in-a-litter-is-fed-exactly-one-half-pound-of-a-82321.html?kudos=1
Gmat Club DS (2) Piglet A was fed exactly 1/6 of the barley today This 1/n must lie between 1/4 and 1/6. Only 1/5 lies between 1/4 and 1/6 (such that n is a positive integer). Hence n must be 5. C
A certain salesman's yearly income is determined by a base salary plus a commission on the sales he makes during the year. Did the salesman's base salary account for more than half of the salesman's yearly income last year?

Given: {Income}={salary}+{commission}. Question basically asks: is {salary}>{commission}?

(1) If the amount of the commission had been 30 percent higher, the salesman's income would have been 10 percent higher last year --> 1.1({salary}+{commission})={salary}+1.3{commission} --> {salary}=2{commission} --> {salary}>{comm

(2) The difference between the amount of the salesman's base salary and the amount of the commission was equal to 50 percent of the salesman's
The difference
base salary
between
last year
X and
-->Y|{salary}-{commission}|=0.5{salary},
= |x-y| cuse we dont know which one notice
is bigger.
that {salary}-{comm
A certain salesman's yearly income is determined by a base salary plus a commission on the sales he makes during the year. Did the salesman's base salary account for
more than half of the salesman's yearly income last year? A. {salary}-{commission}=0.5{salary} --> 0.5{salary}={commission} --> {salary}>{commission}, thus the answer would be YES; The word "difference" implies that one of them IS bigger, but we don't know
Or: which one.
(1) If the amount of the commission had been 30 percent higher, the salesman's income would have been 10 percent higher last year. A. {commission}-{salary}=0.5{salary} --> 1.5{salary}={commission} --> {salary}<{commission}, thus the answer would be No.
Not sufficient. absolute value sign ||, meaning that we can have two cases:
(2) The difference between the amount of the salesman's base salary and the amount of the commission was equal to 50 percent of the salesman's base salary last A. X-Y
um/a-certain-salesman-s-yearly-income-is-determined-by-a-base-126533.html
Gmat Club DS year. Answer: A. A B Y-X
when you combine both the statements together, you get the value of n t be less than -5. However, we know that n denotes the number of set of books and thus, cannot be negative. So, the answer does not stand. However, what this n

Secondly, while evaluating the first statement you got c=200 because you considered all three sets of encyclopedias to be above the sales target of n sets. However, there are three possibilities.

Laura sells encyclopaedias, and her monthly income has two components, a fixed component of $1000, and a variable component of $C for each set of encyclopaedias 1. All three of them are above the sales target in which case 3c=600 implying c=200
that she sells in that month over a sales target of n sets, where n>0. How much did she earn in March? 2. Two of them are above the sales target in which case 2c=600 implying c=300
3. Only one of them is above the sales target in which case c=600
(1) If Laura had sold three fewer sets in March, her income for that month would have been $600 lower than it was.
laura-sells-encyclopaedias-and-her-monthly-income-has-two-64646.html?kudos=1
Gmat Club DS (2) If Laura had sold 10 sets of encyclopaedias in March, her income for that month would have been over $4000. Since, one is already proved to be wrong, the answer has to be one out of 2 and 3. Substituting c=300 in the statement 2 gives us a Cvalue of n=0. Hence, not possible. The answer is the c=600 and hence we need to use this value to work
When Mrs. T's students answer the bonus question correctly, she awards a bonus. If the base score is between 10 and 99, the bonus is equal to 2 times the tens digit Given: Score is a two digit number - S=10x+y (as the score was between 10 and 99);
in the base score. The last test Mrs. T scored was between 10 and 99, and the student answered the bonus question correctly. Was the bonus given greater than 17% Bonus - B=2x
of the base score? Question: is 2x>17/100(10x+y) ? --> is 30x>17y
when it is about digits write down the number as:
(1) The base score of the test was between 50 and 90. (1) The base score of the test was between 50 and 90 --> 5x9 --> the range of 30x is 15030x270 and the range of 17y is 017y153 (as yy can be from 0 to 9). Hence 30x may or may not be more than 17y. Not sufficient.
/if-the-base-score-is-between-10-and-99-the-bonus-is-equal-95183.html?kudos=1
Gmat Club DS (2) Mrs. T added 16 bonus points to the last test she graded. (2) Mrs. T added 16 bonus points to the last test she graded --> B=2x=16B=2x=16 --> x=8x=8 --> 30x=24030x=240. Max value of 17y=15317y=153
B S=10x+y
(as max value of yy is 9), hence 30x30x is more than 17y17y. Sufficient.
If he puts 9 coins in each bag then 7 coins are left over --> c=9q+7, so # of coins can be: 7, 16, 25, 34, 43, 52, 61, ...
A person inherited few gold coins from his father. If he put 9 coins in each bag then 7 coins are left over. However if he puts 7 coins in each bag then 3 coins are left If he puts 7 coins in each bag then 3 coins are left over --> c=7p+3, so # of coins can be: 3, 10, 17, 24, 31, 38, 45, 52, 59, ...
over. What is the number of coins he inherited from his father.
General formula for cc based on above two statements will be: c=63k+52 (the divisor should be the least common multiple of above two divisors 9 and 7, so 63 and the remainder should be the first common integer in above two pat
(1) The number of coins lies between 50 to 120.
forum/a-person-inherited-few-gold-coins-from-his-father-if-he-132500.html
Gmat Club DS (2) If he put 13 coins in each bag then no coin is left over and number of coins being lesser than 200. c=63k+52c=63k+52 means that # of coins can be: 52, 115, 178, 241, ... B

On Jane's credit card account, the average daily balance for 30-day billing cycle is the average (arithmetic mean) of the daily balances at the end of http://gmatclub.com/forum/on-jane-s-credit-card-account-the-average-daily-balance-for-106680.html?kudos=1
each of the 30 days. At the beginning of a certain 30-day billing cycle, Jane's credit card account had a balance of $600. Jane made a payment of
$300 on the account during the billing cycle. If no other amounts were added to or subtracted from the account during the billing cycle, what was the Long question stem... Though try to identify that the question is just testing you on weighted averages. For some days the account had $600, for rest of the days, the account had $300 in it. We need to find the average of the amount in
average daily balance on Jane's account for the billing cycle?
Stmnt 1 - We know now that the account had $600 for 20 days and $300 for the rest of the 10 days. You can easily find the average. Sufficient.
(1) Jane's payment was credited on the 21st day of the billing cycle. You don't need to since it is a DS question, but if you had to find the average, then Avg = (600*20 + 300*10)/30 = 500

Gmat Club DS (2) The average daily balance through the 25th day of the billing cycle was $540. Stmnt 2 - Average for first 25 days = 540 so you can find for how many of these 25 days, the account balance was 600 and for how D
many was it 300. The rest of the 5 days, the balance was of course $300. Hence sufficient.
A car dealership carries only sedans and SUVs, and on Tuesday it sold 1/6 of the sedans that it had in stock at the beginning of the day. If no new
inventory arrived at any point on Tuesday, and the only change in inventory was that some vehicles were sold, did the dealership have more than 100
vehicles in inventory at the beginning of the day Tuesday?
From F.S 1, as the # of cars sold can only assume integral values, we know that the # of SUV's sold at the end of the day were of the form 9k(k is a a positive integer).
(1) By the end of the day, the dealership had sold 8/9 as many sedans as SUVs. Thus, the # of sedans sold at the end of the day = 899k899k = 8k = This represents 16th16thof the total sedans present at the start of the day. Thus, the # of sedans at the start of the day = 8k*6 = 48k. The no of SUV's sold can be minim

forum/a-car-dealership-carries-only-sedans-and-suvs-159268.html?kudos=1
Gmat Club DS (2) The dealership sold 85% as many sedans on Tuesday as it did on Wednesday. From F.S 2, Let the # of sedans sold on Wednesday be S. We know that # of Sedans sold on Tuesday = 85% of S = 1720S1720S = The
b minimum value can be when S=20, i.e. the # of sedans sold on Tuesday = 17. Also, as this is 16th16th
rate = 1 + x

he number of antelope in a certain herd increases every year at a constant rate. If there are 500 antelope in the herd today, how many years will it A , %20 increases means A* (1+0.2)
take for the number of antelope to double?
Growth at some rate means that we have exponential growth. A* x
(1) Ten years from now, there will be more than ten times the current number of antelope in the herd. Growth at some constant amount means linear growth (for example if we were
told that "the number of antelope in a certain herd increases every year by
orum/the-number-of-antelope-in-a-certain-herd-increases-every-99810.html
Gmat Club DS (2) If the herd were to grow in number at twice its current rate, there would be 980 antelope in the group in two years. b 1,000"). A + x

Anda mungkin juga menyukai